Anda di halaman 1dari 31

https://telegram.me/UPSCMaterials https://telegram.me/FreeUPSCMaterials https://telegram.

me/MaterialforExam

PRESTORMINGTM - TEST 4 – ECONOMICS - I - EXPLANATION KEY


1. Ans: (b)
Explanation:
 Commercial banks accept deposits from the public and lend out this to interest earning investment
projects. The rate of interest offered by the bank to the deposit holders is called the ‘borrowing rate’
and the rate at which banks lend out their reserves to investors is called the ‘lending rate’. The
difference between the two rates, called ‘spread’, is the profit that is appropriated by the banks.
 Deposits are broadly of two types – demand deposits, payable by the banks on demand from the
account holder, e.g. current and savings account deposits, and time deposits, which have a fixed period
to maturity, e.g. fixed deposits. Lending by commercial banks consists mainly of cash credit, demand and
short-term loans to private investors and banks’ investments in government securities and other
approved bonds.

2. Ans: (d)
Explanation:
 Gross domestic savings is steadily increasing from 2012.
 2011-12 – INR 30268 bn
 2012-13 – 33692
 2013 -14 – 36081
 2014 -15 – 40199
 2015 -16 – 43019
 As a percentage to GDP, Gross domestic savings shows steady decline.
 Gross capital formation is also steadily increasing from 2012.
 2012-13 – 2307 ( INR billions)
 2013 -14- 2428
 2014-15 – 2619
 2015-16 – 2707
 2016-17 – 3013

3. Ans: (b)
Explanation:
 Gross Domestic product (GDP) measures the aggregate production of final goods and services taking
place within the domestic economy during a year. Here the produce of resident citizens as well as
foreign nationals who reside within that geographical boundary is considered.
 India become the world's sixth-biggest economy, pushing France into seventh place, according to
updated World Bank figures for 2017 release. India's gross domestic product (GDP) amounted to $2.597
trillion at the end of last year, against $2.582 trillion for France. India could soon emerge as the world’s
fifth largest economy if it continues to maintain its current pace of growth, according to Finance Minister
Arun Jaitley.
 The total value of the total output or production of final goods and services produced by the nationals of
a country during a given period of time, generally one year is called Gross National Product (GNP). This
excludes the income of the foreign nationals who reside within the geographical boundary of a country
and includes the income of all residents and non-resident citizens of a country. For example, a citizen of
India working in Dubai may be earning her wage in Dubai but it will be included in India’s GDP.
 GNP = GDP + Factor income earned by the domestic factors of production employed in the rest of the
world – Factor income earned by the factors of production of the rest of the world employed in the
domestic economy
Hence, GNP = GDP + Net Factor Income from Abroad (NFIA)

4. Ans: (a)
Explanation:
 Now, frequently PMI data are quoted about the condition of Indian economy. PMI stands for ‘Purchasing
Managers’ index’ and is considered as an indicator of the economic health and investor sentiments
https://telegram.me/UPSCMaterials https://telegram.me/FreeUPSCMaterials https://telegram.me/MaterialforExam

about the manufacturing sector (there is services PMI as well). In a PMI data, a reading above 50
indicates economic expansion, while a reading below 50 points shows contraction of economic activities.
 Purchasing Manager’s Index predicts the level of industrial production in advance. This is done by
surveying purchasing executives over 500 manufacturing companies in India. The PMI is constructed
separately for manufacturing and services sector. But the manufacturing sector holds more importance.
 For India, the PMI Data is published by Japanese firm Nikkei but compiled and constructed by Markit
Economics (for the US, it is the ISM). A manufacturing PMI and a services PMI are prepared and
published by the two. The Nikkei and markit economics websites says that PMI data are based on
monthly surveys of carefully selected companies.
 The variables used to construct India’s PMI are: Output, New Orders, Employment, Input Costs, Output
Prices, Backlogs of Work, Export Orders, Quantity of Purchases, Suppliers’ Delivery Times, Stocks of
Purchases and Stocks of Finished Goods. Similar variables (but less in number) are used for the
construction of services PMI.
 The PMI is very closely watched, as it shows the investor sentiment in an economy’s manufacturing
sector. In terms of composition, we can say the PMI is a sentiment tracking index. On the other hand,
the Index of Industrial Production indicates changes in production volume or output.

5. Ans: (c)
Explanation:
 Controller General of Accounts (CGA), is a body under the Department of Expenditure, Ministry of
Finance, is the Principal Accounting Adviser to Government of India and is responsible for establishing
and maintaining a technically sound Management Accounting System.
 The Office of CGA prepares monthly and annual analysis of expenditure, revenues, borrowings and
various fiscal indicators for the Union Government. Under Article 150 of the Constitution, the Annual
Appropriation Accounts (Civil) and Union Finance Accounts are submitted to Parliament on the advice of
Comptroller and Auditor General of India. Along with these documents, an M.I.S Report titled 'Accounts
at a Glance’ is prepared and circulated to Hon’ble Members of Parliament.
 It further formulates policies relating to general principles, form and procedure of accounting for the
Central and State Governments. Administer the process of payments, receipts and accounting in Central
Civil Ministries/ Departments. Prepares, consolidates and submits the monthly and annual accounts of
the Central Government through a robust financial reporting system aimed at effective implementation
of the Government fiscal policies.
 It also administers banking arrangements for disbursements of Government expenditures and collection
of government receipts and interacts with the Central Bank for reconciliation of cash balances of the
Union Government.

6. Ans: (a)
Explanation:
 The Department of Financial Services (DFS) oversees several key programs/initiatives and reforms of the
Government concerning the Banking Sector, the Insurance Sector and the Pension Sector in India.
Initiatives and reforms relating to Financial Inclusion, Social Security, and Insurance as a Risk Transfer
mechanism; Credit Flow to the key sectors of the economy/ farmers/ common man are some of the key
focus areas being dealt by the Department. The key flagship schemes being currently run/managed by
the Department include the Pradhan Mantri Jan DhanYojana (PMJDY), Pradhan Mantri Suraksha Bima
Yojana (PMSBY), Pradhan Mantri Jeevan Jyoti Bima Yojana (PMJJBY), Pradhan Mantri Mudra Yojana
(PMMY), Atal Pension Yojana (APY), Pradhan Mantri Vaya Vandana Yojana (PMVVY) and the Stand Up
India Scheme.
 The Department provides policy support to the Public Sector banks (PSBs), Public Sector Insurance
Companies (PSICs) and Development Financial Institutions (DFIs) like National Bank for Agriculture and
Rural Development (NABARD), Small Industries Development Bank of India (SIDBI), India Infrastructure
Finance Company Ltd. (IIFCL), National Housing Bank (NHB), Export-Import Bank of India (EXIM Bank),
Industrial Finance Corporation of India (IFCI). It also monitors the performance of these PSBs, PSICs and
DFIs and undertakes policy formulation in respect of the Banking and Insurance Sector in India. This
Department deals with legislative and policy issues pertaining to the concerned regulatory bodies i.e. the
https://telegram.me/UPSCMaterials https://telegram.me/FreeUPSCMaterials https://telegram.me/MaterialforExam

Reserve Bank of India (RBI), the Insurance Regulatory and Development Authority of India (IRDAI) and
the Pension Fund Regulatory and Development Authority (PFRDA). DFS also deals with the legislative
framework relating to debt recovery.
 Matters relating to International Banking relations are also dealt by the Department.
 Annual Financial Statement (Budget) is prepared by the Department of Economic Affairs, Ministry of
Finance.

7. Ans: (c)
Explanation:
 The all India index of Industrial Production (IIP) is a composite indicator that measures the short-term
changes in the volume of production of a basket of industrial products during a given period with respect
to that in a chosen base period
 The Central Statistics Office (CSO), Ministry of Statistics and Programme Implementation, revises the
base year of the macroeconomic indicators, as a regular exercise, to capture structural changes in the
economy and improve the quality and representativeness of the indices. In this direction, the base year
of the all-India Index of Industrial Production (IIP) has also been revised from 2004-05 to 2011-12 to not
only reflect the changes in the industrial sector but to also align it with the base year of other
macroeconomic indicators like the Gross Domestic Product (GDP), Wholesale Price Index (WPI). IIP in the
revised series will continue to represent the Mining, Manufacturing and Electricity sectors.
 As compared to 2004-05 series, many items were introduced or deleted in the 2011-12 series. For
instance, refined palm oil, cement clinkers and surgical accessories were introduced. Gutka, tooth
brushes, leather shoes, fans, calculators, pens and watches were deleted. In all, 149 new items were
added in the new IIP 2011-12 data series, while 124 of them were deleted. At the broad level, the new
series had 809 items from the manufacturing sector as against 620 from the old 2004-05 series.
Base year 2011-12 Base year 2004-05
Sector
Weights (%) Item groups Weights (%) Item groups
Mining 14.373 1 14.157 1
Manufacturing 77.633 405 75.527 397
Electricity 7.994 1 10.316 1
Total 100 407 100 399

8. Ans: (c)
Explanation:
Wholesale Price Index (WPI)
 The WPI measures the price of a representative basket of wholesale goods. WPI captures price
movements in a most comprehensive way.
 Base Year: 2011-12, Number of Items: 697 grouped into primary articles, Fuel & Power and
manufactured products.
 It is calculated by – Office of the Economic Adviser, DIPP (Department of Industrial Policy and
Promotion), In Ministry of Commerce & Industry.
 It is released on three duration basis.
a. Weekly: on Every Thursday for Primary Articles and Fuel Group.
b. Monthly: On 14th of Every month for all commodities.
c. Final:Final list is released every two months (~EIGHT weeks). When they get authentic price data
for all commodities.
Consumer Price Index (CPI)
 Consumer Price Index is a measure of change in retail prices of goods and services consumed by defined
population group in a given area with reference to a base year. This basket of goods and services
represents the level of living or the utility derived by the consumers at given levels of their income,
prices and tastes. The consumer price index number measures changes only in one of the factors; prices.
 CPI(Rural), CPI(Urban), CPI(Combined) compiled and released by Central Statiscal Office, MOSPI.
 Base Year: 2011-2012
 Monthly and Annual reports
https://telegram.me/UPSCMaterials https://telegram.me/FreeUPSCMaterials https://telegram.me/MaterialforExam

Gross Domestic Product (GDP)


 Base Year: 2011-12, CSO-MOSPI
 Annual and Quarterly estimation.

9. Ans: (c)
 The Banking Ombudsman Scheme is an expeditious and inexpensive forum for bank customers for
resolution of complaints relating to certain services rendered by banks.
 All Scheduled Commercial Banks, Regional Rural Banks and Scheduled Primary Co-operative Banks are
covered under the Scheme. Recently RBI has extended Banking Ombudsman scheme to all deposit
taking NBFCs.
 The Banking Ombudsman is a senior official appointed by the Reserve Bank of India can receive and
consider any complaint relating to the following deficiency in banking services:
1. non-payment or inordinate delay in the payment or collection of cheques, drafts, bills etc.;
2. non-acceptance, without sufficient cause, of small denomination notes tendered for any purpose,
and for charging of commission in respect thereof;
3. non-acceptance, without sufficient cause, of coins tendered and for charging of commission in
respect thereof;
4. non-payment or delay in payment of inward remittances ;
5. failure to issue or delay in issue of drafts, pay orders or bankers’ cheques;
6. non-adherence to prescribed working hours ;
etc.,
 One can file a complaint before the Banking Ombudsman if the reply is not received from the bank
within a period of one month after the bank concerned has received one's complaint, or the bank rejects
the complaint, or if the complainant is not satisfied with the reply given by the bank.
 The Banking Ombudsman may award compensation not exceeding 1 lakh (One Hundred Thousand) to
the complainant for mental agony and harassment. The Banking Ombudsman will take into account the
loss of the complainant's time, expenses incurred by the complainant, harassment and mental anguish
suffered by the complainant while passing such award.

10. Ans: (a)


Explanation:
 In the new and presently running WPI series, significant improvement in concept, coverage and
methodology has been made. In the updated WPI basket, the number of items has been increased
from 676 to 697.
 The new WPI series does not include indirect taxes in its calculation. This insulates the WPI from the
effect of policy changes related to indirect taxes i.e fiscal policy and thus makes it a more accurate
measurement.
 This also brings new WPI series closer to Producer Price Index and is in consonance with the global
practices. The item level indices are being compiled based on statistically robust Geometric mean as
compared to Arithmetic mean used in the WPI 2004-05 series.

11. Ans: (b)


Explanation:
 National Bank for Agriculture and Rural Development (NABARD) is the refinancing agency. It does not
provide direct credit to rural households. All other banks like Public sector banks, Private sector banks
and cooperative banks will provide credit to rural households.

12. Ans: (d)


Explanation:
Government of India created the RIDF in NABARD in 1995-96, with an initial corpus of Rs.2,000 crore. With
the allocation of Rs.28,000 crore for 2018-19 under RIDF XXIV, the cumulative allocation has reached
Rs.3,20,500 crore, including Rs. 18,500 crore under Bharat Nirman.
Eligible Activities
https://telegram.me/UPSCMaterials https://telegram.me/FreeUPSCMaterials https://telegram.me/MaterialforExam

At present, there are 36 eligible activities under RIDF as approved by GoI. (Annexure-I). The eligible activities
are classified under three broad categories i.e.
 Agriculture and related sector
 Social sector
 Rural connectivity
Eligible Institutions
 State Governments / Union Territories
 State Owned Corporations / State Govt. Undertakings
 State Govt. Sponsored / Supported Organizations
 Panchayat Raj Institutions/Self Help Groups (SHGs)/ NGOs
 (provided the projects are submitted through the nodal department of State Government, i.e. Finance
Department)
 Domestic commercial banks also contribute to the Fund to the extent of their shortfall in stipulated
priority sector lending to agriculture.

13. Ans: (d)


Explanation:
 Automated Teller Machine is a computerized machine that provides the customers of banks the facility of
accessing their account for dispensing cash and to carry out other financial & non-financial transactions
without the need to actually visit their bank branch.
 Basically there are three types of ATMs in India. ATMs which are owned by banks, brown label ATMs and
white label ATMs
 ATMs by nature of installation: Bank ATMs: the ATMs which are owned, managed, installed by banks. The
most common you see in bank branch buildings.
 Brown – label ATMs: they are little different from bank ATMs , they are outsourced to a company, who
manages, installs and look after the ATMs. However these has a logo of the bank in it, which makes sure,
that ATM is installed by the bank. Mostly these were used by private sector banks in India.
 White – lable ATMs: white label ATMs are similar to brown label ATMs except the fact, they don’t have any
bank logo in them. Now, tataindicash , currently famous WLATM, do have a logo of indicash in it.

14. Ans: (b)


Explanation:
 e-Kuber is the Core Banking Solution of Reserve Bank of India. E-Kuber provides the provision of a single
current account for each bank across the country, with decentralised access to this account from
anywhere-anytime using portal based services in a safe manner.
 Core Banking Solutions (CBS) can be defined as a solution that enables banks to offer a multitude of
customer-centric services on a 24x7 basis from a single location, supporting retail as well as corporate
banking activities, as well as all possible delivery channels existing and proposed. Using CBS, customers
can access their accounts from any branch, anywhere, irrespective of where they have physically opened
their accounts. Almost all branches of commercial banks, including the Regional Rural Banks (RRBs), are
brought into the core-banking fold.
 e-Kuber enables ease of operations. The system also benefits state /central Governments as users. Some
of the facilities offered include the provision of portal based access which allows Government
departments to access on anywhere-anytime basis and view their balances – of all types including the
Ways and Means Advances, drawings, funds positions and the like – all in a consolidated manner so as to
help them in better funds management.
 The e-kuber system can be accessed either through INFINET or Internet. The INFINET is a Closed User
Group Network for the exclusive use of member banks and financial institutions and is the
communication backbone for the National Payments System, which caters mainly to inter-bank
applications like Real Time Gross Settlement (RTGS), Delivery Vs Payment, Government Transactions,
Automatic Clearing House, etc.
 The e-kuber system, implemented in 2012, is reported to be one of the foremost central bank oriented
Core Banking Systems in the world.
https://telegram.me/UPSCMaterials https://telegram.me/FreeUPSCMaterials https://telegram.me/MaterialforExam

 Auction of Government securities is done through e-kuber system. Sovereign Gold Bonds are available
for subscription at the branches of scheduled commercial banks and designated post offices through
RBI’s e-kuber system. Goods and Service Tax (GST) settlements are also proposed to be done through e-
kuber. On 7 April 2016, RBI launched a platform to enable trading in the priority sector lending
certificates (PSLC) through its Core Banking Solution (CBS) portal (e-Kuber).
 Kuber refers to Lord Kubera – the lord of wealth in Hindu Mythology. Technology partner for RBI for
launching e-kuber is Polaris Ltd.

15. Ans: (c)


Explanation:
 The quality of assets in the Indian banking system has emerged as a cause of concern for the central
bank in recent times. The asset quality has suffered due to a rise in the non-performing loans. Since the
banking and the financial system is highly interconnected, the failure of one bank, or some banks, is
likely to affect the stability of other banks. This interdependence is measured by the Banking Stability
Index.
 The Reserve Bank of India (RBI) defines Banking Stability Index (BSI) as “the expected number of banks
that could become distressed given that at least one bank has become distressed”.
 Further, the Financial Stability Report (FSR) noted that vulnerability for selected banks has gone up as
the toxicity and vulnerability indices have shown co-movement with BSI. The toxicity index is defined as
“the average probability that a bank under distress may cause distress to another bank in the system”,
while the vulnerability index is defined as “the average probability of a bank coming under distress given
distress in other banks in the system”.

16. Ans: (b)


Explanation:
 Fiscal Deficit - The fiscal deficit is the excess of total budget expenditure over total budget receipts
excluding borrowings during a fiscal year. In simple words, it is the amount of borrowing the government
has to resort to meet its expenses.
Fiscal Deficit = revenue deficit + Capital expenditure – Non debt creating capital receipts
Non debt creating capital receipts are those which do not give rise to debt, as the name itself suggests.
For instance, loan recovery and PSU proceeds from disinvestment in the Public Sector Undertakings.
 Primary Deficit - Primary deficit is defined as the fiscal deficit of current year minus interest payments
on previous borrowings. Primary deficit excludes the burden of previous debt and only shows the net
increase in the government’s Debt that is due in the current fiscal year. Therefore, a reduction in the
primary deficit means the government is following policies to bridge the fiscal gap during the financial
year.
 Deficit Financing - It refers to the method of Government that is followed to meet the excess of
expenditure over income in its budget. Generally, deficit financing can be achieved through borrowings
from market, borrowing from the RBI or drawing from the government cash balance held by the RBI.
 Budget deficit - Budget deficit is the overall type of deficit. It means the excess of total expenditure over
total revenues. Budget deficit includes both capital and the revenue items mentioned in the receipts and
expenditure. The term ‘deficit financing’ is used for filling this deficit only. The Budgetary deficit is
financed either by borrowings, taxation, or printing money. But Governments have largely relied on
borrowings for financing the budget deficit, hence, giving rise of Government Debt.

17. Ans: (d)


Explanation:
 Cess is a tax on tax, levied by the government for specific purpose.
 Demerit or sin goods are, as the name suggests, in economics, demerit goods are "goods or services
whose consumption is considered unhealthy, degrading, or otherwise socially undesirable due to the
perceived negative effects on consumers themselves". These are usually over-consumed if left to market
forces.
https://telegram.me/UPSCMaterials https://telegram.me/FreeUPSCMaterials https://telegram.me/MaterialforExam

 Under the GST regime, a cess is levied on luxury, demerit and sin goods to make good the loss suffered
by the states on account of roll out of the new indirect tax regime. This is levied on top of the highest tax
rate of 28% on these goods.

18. Ans: (c)


Explanation:
 The Society for Worldwide Interbank Financial Telecommunication (SWIFT) provides a network that
enables financial institutions worldwide to send and receive information about financial transactions in
a secure, standardized and reliable environment.
 Society for Worldwide Interbank Financial Telecommunications (SWIFT) is member-owned cooperative
that provides safe and secure financial transactions for its members. Established in 1973, SWIFT uses a
standardized proprietary communications platform to facilitate the transmission of information about
financial transactions. Financial institutions securely exchange this information, including payment
instructions, among themselves.
 SWIFT neither holds funds on its own nor manages external client accounts. The cooperative began
operating in 15 countries in 1973 and now operates in 210 countries, linking more than 10,000 financial
institutions.
 SWIFT is headquartered in Belgium and has offices in the United States, Brazil, Australia, India, Japan,
Korea, Austria, Belgium, France, Germany, Italy, South Africa, Spain, Sweden, Switzerland, the United
Kingdom, UAE and Russian Federation.
 For money transfers, SWIFT assigns each participating financial organization a unique code with either
eight or eleven characters. The code has three interchangeable names: the bank identifier code (BIC),
SWIFT code, SWIFT ID, or ISO 9362 code.

19. Ans: (c)


Explanation:
 The Financial Sector Assessment Program (FSAP) is a joint program of the International Monetary
Fund and the World Bank. Launched in 1999 in the wake of the Asian financial crisis, the program brings
together Bank and Fund expertise to help countries reduce the likelihood and severity of financial sector
crises. The FSAP provides a comprehensive framework through which assessors and authorities in
participating countries can identify financial system vulnerabilities and develop appropriate policy
responses.
 The program also helps bring financial sector analysis closer to the center of economic policy discussions
within a country and with the Fund and the Bank. Participating in the program helps inform domestic
policy-makers of the need for sequenced actions in areas requiring urgent attention and offers countries
a comprehensive framework in which to take on financial sector reforms. It also provides countries with
an opportunity to measure their compliance with financial sector standards and codes and, therefore, to
benchmark their regulatory and supervisory systems against internationally-accepted practices.
 The FSAP follows a three-pronged approach when looking at the country’s financial sector:
 The soundness of a financial system versus its vulnerabilities and risks that increase the likelihood or
potential severity of financial sector crises.
 A country’s developmental needs in terms of infrastructure, institutions and markets.
 A country’s compliance with the observance of selected financial sector standards and codes.

20. Ans: (c)


Explanation:
 Regional Rural Banks were established under the provisions of an Ordinance passed in September 1975
and the RRB Act 1976 to provide sufficient banking and credit facility for agriculture and other rural
sectors. These were set up on the recommendations of The Narshimman committee Working Group
during the tenure of Indira Gandhi's government with a view to include rural areas into economic
mainstream since that time about 70% of the Indian Population was of Rural Orientation.
 The RRBs were owned by three entities with their respective shares as follows: Central Government →
50% State government → 15% Sponsor bank → 35%
 Regional Rural Banks are regulated by National Bank for Agriculture and Rural Development (NABARD).
https://telegram.me/UPSCMaterials https://telegram.me/FreeUPSCMaterials https://telegram.me/MaterialforExam

 They have been created with a view of serving primarily the rural areas of India with basic banking
and financial services. However, RRBs may have branches set up for urban operations and their area of
operation may include urban areas too.

21. Ans: (d)


Explanation:
 Pradhan Mantri MUDRA Yojana (PMMY) is a scheme launched by the Hon’ble Prime Minister on April 8,
2015 for providing loans upto 10 lakh to the non-corporate, non-farm small/micro enterprises. These
loans are classified as MUDRA loans under PMMY.
 These loans are given by Commercial Banks, RRBs, Small Finance Banks, Cooperative Banks, MFIs and
NBFCs. The borrower can approach any of the lending institutions mentioned above or can apply online
through this portal.
 Under the aegis of PMMY, MUDRA has created three products namely 'Shishu', 'Kishore' and 'Tarun' to
signify the stage of growth / development and funding needs of the beneficiary micro unit /
entrepreneur and also provide a reference point for the next phase of graduation / growth.
 Those eligible to borrow from MUDRA bank are
 Small manufacturing unit
 Shopkeepers
 Fruit and vegetable vendors
 Artisans

22. Ans: (c)


Explanation:
 Information utility is an information network which would store financial data like borrowings, default
and security interests among others of firms. The utility would specialise in procuring, maintaining and
providing/supplying financial information to businesses, financial institutions, adjudicating authority,
insolvency professionals and other relevant stake holders.
 Information utilities are governed by the Insolvency and Bankruptcy code 2016 and IBBI (Information
Utilities) Regulations 2017. The Insolvency and Bankruptcy Board of India (IBBI) overseas aspects such as
registration and cancellation of these entities, their shareholding and governance among others.
Recently, IBBI eased norms for information utilities, allowing Indian firms listed on stock exchanges to
hold 100% in such firms. It also allowed individuals to hold 51% in the utility for a period of three years.
 The objective behind information utilities is to provide high-quality, authenticated information about
debts and defaults, as per the report of the Working Group on Information Utility published by the
Ministry of Corporate Affairs. Information utilities are expected to play a key role as they allow storage
of financial information of registered users and expeditiously process and verify information received.
 Moreover, the database and records maintained by them would help lenders in taking informed
decisions about credit transactions. It would also make debtors cautious as credit information is
available with the utility. More importantly, information available with the utility can be used as
evidence in bankruptcy cases before the National Company Law Tribunal.
 National e-Governance Services Ltd (NeSL) became India’s first information utility (IU) for bankruptcy
cases under the Insolvency and Bankruptcy Code 2016. NeSL is owned by State Bank of India and Life
Insurance Corporation Ltd., among others. Recently, the Insolvency and Bankruptcy Board of India (IBBI)
eased ownership norms for setting up such utilities.

23. Ans: (b)


Explanation: The three types of taxes are the proportional tax, the progressive tax, and the regressive tax.
 A proportional tax imposes the same percentage of taxation on everyone, regardless of income. If the
percentage tax rate is constant, the average tax rate is constant, regardless of income. This means that if
a person's income goes up, the percentage of total income paid in taxes doesn't change.
 The second tax, the progressive tax, imposes a higher percentage rate of taxation on those with higher
incomes. Progressive taxes use a marginal tax rate that increases as the amount of taxable income
increases. Therefore, the percentage of income paid in taxes increases as income goes up.Progressive
taxes are imposed in an attempt to reduce the tax incidence of people with a lower ability to pay, as
https://telegram.me/UPSCMaterials https://telegram.me/FreeUPSCMaterials https://telegram.me/MaterialforExam

such taxes shift the incidence increasingly to those with a higher ability-to-pay. Progressive tax rates,
while raising taxes on high income, have the goal and corresponding effect of reducing the burden on
low income, improving income equality
 The final tax is the regressive tax, which imposes a higher percentage rate of taxation on low incomes
than on high incomes. For example, if the state sales tax were 5%, the person with the lower income
would pay a greater percentage of their total income in sales tax.In terms of individual income and
wealth, a regressive tax imposes a greater burden (relative to resources) on the poor than on the rich:
there is an inverse relationship between the tax rate and the taxpayer's ability to pay, as measured by
assets, consumption, or income. These taxes tend to reduce the tax burden of the people with a
higher ability to pay, as they shift the relative burden increasingly to those with a lower ability to pay.

24. Ans: (c)


Explanation:
 Deficit financing is a method of meeting government deficits through the creation of new money. The
deficit is the gap caused by the excess of government expenditure over its receipts.
 Creation of new money to meet the deficit in use for a long time. But it has now being given up. Instead
a new scheme called ways and Means Advances is being ushered in with effect from April 1997. Under
this system the government can get only temporary loans from RBI to overcome the mismatch between
its receipts and expenditures.
 The leading deficit indicator and also the best one to measure the health of the budget in the Indian
context is fiscal deficit. The fiscal deficit represents borrowing by the government. This borrowing is
made by the government mostly from the domestic financial market by issuing bonds or treasury bills.

25. Ans: (a)


Explanation:
 Rupee depreciation is when rupee value decreases (becomes less expensive) and more rupees required
to buy one unit of foreign currency. This is also known as weakening of rupee as now INR worth is less
than foreign currency. More demand for foreign currency.
 Consequences :
i. A weak rupee would make imports of goods and services costlier as the importers have to pay more
than earlier. The firms will pass on increased import costs to the end-consumers.
ii. Exporters will have higher revenue. This increases the export of goods and services.
iii. For each dollar taken abroad for spending, the traveller has to pay more and thus his trip will
become costlier. So a huge burden for students who wish to study abroad.

26. Ans: (a)


Explanation:
 Inflation is the persistent and appreciable rise in the general level of prices.
 Measures to reduce inflation :
A. Monetary measures: Bank rate policy, variable reserve ratio, liquidity adjustment facility,
Demonetisation of currency, Issue of new currency.
 The Central bank could increase interest rates. Higher rates make borrowing more expensive and saving
more attractive. This should lead to lower growth in consumer spending and investment.
 A higher interest rate should also lead to higher exchange rate, which helps to reduce inflationary
pressure by
 Making imports cheaper.
 Reducing demand for exports and
 Increasing incentive for exporters to cut costs.
B. Fiscal measures: Reduction in government expenditure, Increase in Direct taxes, Decreasing Indirect
taxes, Reduction in government borrowing.
 The government can increase taxes (such as income tax and VAT) and cut spending. This improves the
budget situation and helps to reduce demand in the economy.
 Both these policies reduce inflation by reducing the growth of Aggregate Demand. If economic growth is
rapid, reducing growth of AD can reduce inflationary pressures without causing a recession.
https://telegram.me/UPSCMaterials https://telegram.me/FreeUPSCMaterials https://telegram.me/MaterialforExam

a. Trade Measures : Increasing imports to meet the supply shortage.


b. Administrative measures

27. Ans: (d)


Explanation:
 Cash Reserve Ratio: Scheduled commercial banks are required to keep certain percentage of their Net
Demand and Time Deposits with RBI under RBI Act of 1934. If this reserve ratio is increased, the banks
have to deposit more money with RBI. So, the resource available to banks for lending will come down.
The money supply will come down.
 Statutory Liquidity Ratio: Scheduled commercial banks are required to keep certain percentage of their
net time and demand deposits in their vault itself. It need not be deposited with RBI. This reserve is a
precautionary measure. It prevents banks from lending all its deposits which is too risky. Like CRR,
increasing SLR will decrease the money supply and vice versa.
 Repo rate and Reverse repo rate: These are liquidity adjustment facility by RBI and is a short term credit
control measure. It is to absorb the excess liquidity in the economy. Repo rate is the rate at which
commercial banks borrow from RBI by mortgaging their dated Government securities and Treasury bills.
So if Repo is increased, banks either to reduce the borrowing from RBI or borrow at higher rate and
charge higher interest rate among customers. So it decreases money supply. If the repo rate is
decreased, the reverse will be the case.
 Reverse repo rate is the rate at which RBI borrows from commercial banks by mortgaging its dated
government securities and treasury bills.

28. Ans: (c)


Explanation:
 An ETF, or exchange-traded fund, is a marketable security that tracks a stock index, a commodity, bonds,
or a basket of assets. An ETF is a type of fund that owns underlying assets (shares of stock, bonds, oil
futures, gold bullion, foreign currency, etc.) and divides ownership of those assets into shares.
 Although similar in many ways, ETFs differ from mutual funds because shares trade like common stock
on an exchange. The price of an ETF’s shares will change throughout the day as they are bought and
sold. The largest ETFs typically have higher average daily volume and lower fees than mutual fund shares
which makes them an attractive alternative for individual investors.
 While most ETFs track stock indexes, there are also ETFs that invest in commodity markets, currencies,
bonds, and other asset classes. Many ETFs also have options available for investors to use income,
speculation, or hedging strategies.

29. Ans: (c)


Explanation:
 Bharat Bill Payment System (BBPS) is an integrated bill payment system in India offering interoperable
and accessible bill payment service to customers through a network of agents of registered member as
Agent Institutions (AI) enabling multiple payment modes, and providing instant confirmation of
payment.
 It is an integrated and interoperable bill payment service for customers across geographies. It offers
reliability and safety of the transactions. You can pay your bills using BBPS from anywhere anytime.
Moreover, it allows you to pay bills through networks of agents or online. You get multiple payment
modes with instant confirmation of the payment. With BBPS, you can pay your bill using electronic
transfer instead of pay in cash. Hence, it is said to be a big step towards a less-cash society.
 National Payments Corporation of India (NPCI) functions as the authorised Bharat Bill Payment Central
Unit (BBPCU), which will be responsible for setting business standards, rules and procedures for
technical and business requirements for all the participants. NPCI, as the BBPCU, will also undertake
clearing and settlement activities related to transactions routed through BBPS.
 Existing bill aggregators and banks are envisaged to work as Operating Units to provide an interoperable
bill payment system irrespective of which unit has on-boarded a particular biller.
 Moreover, Bharat Bill Payment System has a wide variety of payment options. You can make full, excess,
part, penalty and multiple payments with it. It also gives you many payment modes to choose from.
https://telegram.me/UPSCMaterials https://telegram.me/FreeUPSCMaterials https://telegram.me/MaterialforExam

These include cash, credit/debit cards, IMPS, internet banking, NEFT, prepaid card, wallet and UPI. You
can choose any of these as per your convenience to pay your bill.

30. Ans: (b)


Explanation:
 National Investment and Infrastructure Fund (NIIF) is a fund created by the Government of India for
enhancing infrastructure financing in the country. This is different from the National Investment Fund.
NIIF was proposed to be set up as a Trust, to raise debt to invest in the equity of infrastructure finance
companies such as Indian Rail Finance Corporation (IRFC) and National Housing Bank (NHB). The idea is
that these infrastructure finance companies can then leverage this extra equity, manifold. In that sense,
NIIF is a banker of the banker of the banker. NIIF got registered with SEBI as Category II Alternative
Investment Fund (AIF).
 The objective of NIIF would be to maximize economic impact mainly through infrastructure development
in commercially viable projects, both Greenfield and brownfield, including stalled projects. It could also
consider other nationally important projects, for example, in manufacturing, if commercially viable.
 The functions of NIIF are as follows:
 Fund raising through suitable instruments including off-shore credit enhanced bonds, and attracting
anchor investors to participate as partners in NIIF;
 Servicing of the investors of NIIF.
 Considering and approving candidate companies/institutions/ projects (including state entities) for
investments and periodic monitoring of investments.
 Investing in the corpus created by Asset Management Companies (AMCs) for investing in private
equity.
 Preparing a shelf of infrastructure projects and providing advisory services.

31. Ans: (c)


Explanation:
 The Green Climate Fund (GCF) approved (March 2018) the proposal submitted by NABARD for the ‘line
of credit for solar rooftop segment for commercial, industrial and residential housing sectors’ with a GCF
loan support of $100 million. The proposal is the first private sector facility proposal sanctioned by GCF
to India and will be executed through Tata Clentech Capital (TCCL).
 NABARD has been accredited as the direct access entity (DAE) of the Green Climate Fund for
channelizing resources under this fund. It aims to use GCF resources for projects and programmes aimed
at climate-resilient and low-emission development. The GCF has been designated as an operating entity
of the financial mechanism of the UNFCCC and aims to support developing countries to limit or reduce
greenhouse gas emissions and to adapt to the impacts of the climate change.
 National Bank for Agriculture and Rural Development (NABARD) has been reaccredited as National
Implementing Entity (NIE) by the Adaptation Fund set up under the Kyoto Protocol of the United Nations
Framework Convention on Climate Change (UNFCCC). NABARD received a communication in this regard
on 12th January 2018. The reaccreditation is till 26 December 2022. NABARD was first accredited as NIE
for Adaptation Fund in July 2012 and is the only NIE for India.

32. Ans: (d)


Explanation:
 Financial inclusion means that individuals and businesses have access to useful and affordable financial
products and services that meet their needs – transactions, payments, savings, credit and insurance –
delivered in a responsible and sustainable way.
 Some financial inclusion schemes are
 Pradhan Mantri Jan Dhan Yojana: Universal access to banking facilities with atleast one basic bank
account to every household, financial literacy, access to credit, insurance and pension facility.
 Pradhan Mantri Jeevan Jyoti Bima Yojana
 Pradhan Mantri Suraksha Bima Yojana
 Atal Pension Yojana
 Pradhan Mantri Mudra Yojana
https://telegram.me/UPSCMaterials https://telegram.me/FreeUPSCMaterials https://telegram.me/MaterialforExam

 Stand up India Scheme


 Pradhan Mantri Vaya Vandhana Yojana

33. Ans: (a)


Explanation:
 The Medium-term Expenditure Framework Statement is a statement presented to the Parliament
under Section 3 of the Fiscal Responsibility and Budget Management (FRBM) Act, 2003 and sets forth a
three-year rolling target for the expenditure indicators with specification of underlying assumptions and
risks involved.
 The statement provides an estimate of expenditure commitments for various items viz. Education,
Health, Rural Development, Energy, Subsidies and Pension etc. While formulating the MTEF Statement,
information on expenditure commitments spread across the various central ministries on salaries
(including grants-in-aid for salaries) and pensions, grants-in-aid for creation of capital assets, major
programme, interest payment, defense expenditure and major subsidies etc. and other commitments of
Government, will be considered. To take an example, in MTEF, salary component which now appears
scattered amongst the various Demand for Grants of central Ministries would be aggregated and
projected into the future. Expenditure commitments are shown separately for Revenue and Capital
expenditure. “Grants-in-aid for creation of capital assets" and its projection are also depicted as a part of
Revenue expenditure.
 The objective of the MTEF is to provide a closer integration between budget and the FRBM Statements.
This Statement is presented separately in the session next to the session in which Union Budget is
presented, i.e. normally in the Monsoon Session.

34. Ans: (b)


Explanation:
 India was ranked at 62nd place among emerging economies on Inclusive Development Index (IDI-2018)
released by World Economic Forum (WEF)
 Inclusive Development Index (IDI) measures progress of 103 economies on three individual pillars –
growth and development; inclusion; and inter-generational equity.
 It has been divided into two parts. The first part covers 29 advanced economies and second 74 emerging
economies. The index takes into account the living standards, environmental sustainability and
protection of future generations from further indebtedness. The index also has classified countries into
five sub-categories in terms of five-year trend of their overall Inclusive Development Growth score —
receding, slowly receding, stable, slowly receding, stable, slowly advancing and advancing.
 It was ranked 60th among 79 developing economies in IDI-2017. India was among ten emerging
economies with ‘advancing’ trend, despites its low overall score. Of three pillars that make up index,
India was ranked 72nd for inclusion, 66th for growth and development and 44th for inter-generational
equity.

35. Ans: (b)


Explanation:
 General anti-avoidance rule (GAAR) is an anti-tax avoidance law under Chapter X-A of the Income Tax
Act, 1961 of India. It is framed by the Department of Revenue under the Ministry of Finance. GAAR was
originally proposed in the Direct Tax Code 2009 and was targeted at arrangements or transactions made
specifically to avoid taxes.
 GAAR provisions were also present in the Direct Tax Code 2010 and Direct Tax Code 2013. However, the
Direct Tax Code did not see the light of the day and was not implemented in India. GAAR was finally
introduced in India vide Finance Act, 2012.
 The regulation allows tax officials to deny tax benefits, if a deal is found without any commercial purpose
other than tax avoidance. It allows tax officials to target participatory notes. Under GAAR, the investor
has to prove that the participatory note was not set to avoid taxes. It also allows officials to deny double
taxationavoidance benefits, if deals made in tax havens like Mauritius were found to be avoiding taxes.
https://telegram.me/UPSCMaterials https://telegram.me/FreeUPSCMaterials https://telegram.me/MaterialforExam

 Tax avoidance is legal; but now, large scale revenue loss is occurring due to aggressive tax planning by
corporate using avoidance opportunities. Governments in many countries are introducing anti-
avoidance rules to check this revenue loss from excessive avoidance.

36. Ans: (c)


Explanation:
 Fiscal Consolidation refers to the policies undertaken by Governments (national and sub-national levels)
to reduce their deficits and accumulation of debt stock.
 Key deficits of government are the revenue deficit and the fiscal deficit. The gains from the economic
reforms introduced in India in early nineties could not be sustained for a much longer period. Deficits
were widening and by 1999-2000 the combined fiscal deficit (of centre and states) almost reached levels
of the crisis year ‘1990-91’. Sustainability of debt too was becoming a major issue. In December 2000,
Government of India introduced the Fiscal Responsibility and Budget Management (FRBM) Bill in the
Parliament as it was felt that institutional support in the form of fiscal rules would help in setting the
agenda for the future fiscal consolidation programme. The Twelfth Finance Commission recommended
in November 2004 that state governments to enact their fiscal responsibility legislations. However,
states like Karnataka, Kerala, Punjab, Tamil Nadu and Uttar Pradesh had already enacted their fiscal
responsibility legislation even before the Commission recommended so.
 Implementation of Fiscal Responsibility and Budget Management (FRBM) legislation at national as well
as at sub-national levels in India during the period 2005-10 helped both the Union and the States to
achieve considerable correction in their respective fiscal position, which was weak prior to 2005.
 Fiscal deficit - The difference between total revenue and total expenditure of the government is termed
as fiscal deficit. It is an indication of the total borrowings needed by the government.
 2012 – 4.93
 2013 – 4.48
 2014 –4.1
 2015 - 3.89
 2016 –3.52

37. Ans: (b)


 TAX BUOYANCY: Tax buoyancy is an indicator to measure efficiency and responsiveness of revenue
mobilization in response to growth in the Gross domestic product or National income. A tax is said to
be buoyant if the tax revenues increase more than proportionately in response to a rise in national
income or output. A tax is buoyant when revenues increase by more than, say, 1 per cent for a 1 per
cent increase in GDP.
 TAX ELASTICITY: The tax incidence depends on the relative price elasticity of supply and demand. When
supply is more elastic than demand, buyers bear most of the tax burden. When demand is
more elastic than supply, producers bear most of the cost of the tax. Tax revenue is larger the more
inelastic the demand and supply are.

38. Ans: (b)


Explanation:
 Base erosion and profit shifting (BEPS) refers to tax avoidance strategies that exploit gaps and
mismatches in tax rules to artificially shift profits to low or no-tax locations.
 Although some of the schemes used are illegal, most are not. This undermines the fairness and integrity
of tax systems because businesses that operate across borders can use BEPS to gain a competitive
advantage over enterprises that operate at a domestic level. Moreover, when taxpayers see
multinational corporations legally avoiding income tax, it undermines voluntary compliance by all
taxpayers.
 BEPS is of major significance for developing countries due to their heavy reliance on corporate income
tax, particularly from multinational enterprises. Engaging developing countries in the international tax
agenda is important to ensure that they receive support to address their specific needs.
 The Inclusive Framework on BEPS brings together over 115 countries and jurisdictions to collaborate on
the implementation of the OECD/ G20 Base Erosion and Profit Shifting (BEPS) Package.
https://telegram.me/UPSCMaterials https://telegram.me/FreeUPSCMaterials https://telegram.me/MaterialforExam

39. Ans: (d)


Explanation:
 Financial Stability and Development Council (FSDC) is an apex-level body constituted by
the government of India. The idea to create such a super regulatory bodywas first mooted by
the Raghuram Rajan Committee in 2008. Finally in 2010, the then Finance Minister of India, decided to
set up such an autonomous body dealing with macro prudential and financial regularities in the entire
financial sector of India.
 An apex-level FSDC is not a statutory body. The recent global economic meltdown has put pressure on
governments and institutions across the globe to regulate their economic assets. This council is seen
as India's initiative to be better conditioned to prevent such incidents in future.
 The new body envisages to strengthen and institutionalize the mechanism of maintaining financial
stability, financial sector development, inter-regulatory coordination along with monitoring macro-
prudential regulation of economy. No funds are separately allocated to the council for undertaking its
activities.
 FSDC was formed to bring greater coordination among financial market regulators. The council is headed
by the finance minister and has the Reserve Bank of India (RBI) governor and chairpersons of the
Securities and Exchange Board of India, Insurance Regulatory and Development Authority and Pension
Fund Regulatory and Development Authority as other members along with finance ministry officials.

40. Ans: (c)


Explanation:
 The low-level equilibrium trap is a concept in economics developed by Richard R. Nelson, in which
at low levels of per capita income people are too poor to save and invest much, and this low level of
investment results in low rate of growth in national income.
 As per capita income rises above a certain minimum level at which there is zero saving, a rising
proportion of income will be saved and invested and this will lead to higher rate of growth in income.
 Getting out of the trap requires increasing the rate of growth of income to the levels higher than the
rate of increase in population.
 In Nelson's opinion following four conditions are conducive to trapping:
 A high correlation between the level of per-capita income and the rate of population growth
 A low propensity to direct additional per-capita income to increasing per-capita investment
 Scarcity of uncultivated arable land
 Inefficient production method

41. Ans: (d)


Explanation:
 The preamble of the Reserve Bank of India describes the basic functions of the reserve bank as:
"to regulate the issue of Bank notes and keeping of reserves with a view to securing monetary stability in
India and generally to operate the currency and credit system of the country to its advantage; to have a
modern monetary policy framework to meet the challenge of an increasingly complex economy, to
maintain price stability while keeping in mind the objective of growth."
 The RBI plays an important part in the Development Strategy of the Government of India.
 Some functions of RBI:
 Regulator and supervisor of Indian Financial system
 Regulator and supervisor of the payment and settlement systems
 Banker and debt manager to government (centre and state)
 Managing foreign exchange
 Issuer of currency
 Banker’s bank – Lender of last resort
 Regulator of the banking system

42. Ans: (b)


Explanation:
https://telegram.me/UPSCMaterials https://telegram.me/FreeUPSCMaterials https://telegram.me/MaterialforExam

 The reserves are managed by the Reserve Bank of India for the Indian government and the main
component is foreign currency assets.
 Foreign exchange reserves act as the first line of defense for India in case of economic slowdown, but
acquisition of reserves has its own costs. Foreign exchange reserves facilitate external trade and
payment and promote orderly development and maintenance of foreign exchange market in India.
 As of October 2018, India has
 $375.23 billion foreign currency assets,
 $20.45 billion gold reserves,
 $2.47 billion in reserve tranche position (reserve tranche is an emergency account that IMF
members can withdraw from without agreeing to conditions or paying a service fee)
 $1.47 billion in SDR (special drawing rights).

43. Ans: (d)


Explanation:
 Launched in December, 1993
 It provides a mechanism for the Members of Parliament to recommend works of developmental nature
for creation of durable community assets and for provision of basic facilities including community
infrastructure, based on locally felt needs
 MPLADS is a centrally-sponsored plan scheme fully funded by the government of India under which
funds are released in the form of grants in-aid directly to the district authorities.
 The funds released under the scheme are non-lapsable. Funds not released in a particular year is carried
forward to the subsequent years, subject to eligibility.
 The elected Lok Sabha Members can recommend works in their respective constituencies.
 The elected members of the Rajya Sabha can recommend works anywhere in the state from which they
are elected.
 Nominated Members of the Lok Sabha and Rajya Sabha can recommend works for implementation,
anywhere in the country.

44. Ans: (b)


Explanation:
 The Special 301 Report is prepared annually by the Office of the United States Trade Representative
(USTR) that identifies trade barriers to United States companies and products due to the intellectual
property laws, such as copyright, patents and trademarks, in other countries.
 The Office of United States (US) Trade Representative (USTR) has once again placed India on ‘Priority
Watch List’ in its annual Special 301 Report on the state of intellectual property protection.
 In 2018 Special 301 Report, the USTR has placed 12 countries on its Priority Watch List. Special 301
Report identifies US trading partners that do not adequately or effectively protect and enforce IP rights
or deny market access to its innovators and creators that rely on protection of their IP rights.
 Among other issues include India’s enforcement action and policies that are insufficient to curb the
problem, copyright policies that do not properly incentivise the creation and commercialisation of
content and an outdated and insufficient trade secrets legal framework.

45. Ans: (c)


Explanation:
 Sterilization is a form of monetary action in which a central bank seeks to limit the effect of inflows and
outflows of capital on the money supply. Sterilization most frequently involves the purchase or sale of
financial assets by a central bank, and is designed to offset the effect of foreign exchange intervention.
The sterilization process is used to manipulate the value of one domestic currency relative to another,
and is initiated in the foreign exchange market.
 Suppose the RBI decides to buy US dollars (USD) from the market. Now, the money held by the RBI does
not form part of the banking system. So, if the RBI releases rupees from its coffers to buy dollars, the
money supply in the banking system increases. That can be a problem.
 Remember the textbook definition of inflation? It is "too much money chasing too few goods". Thus, if
there is no accompanying increase in output, the excess supply of money will "chase" the same quantity
https://telegram.me/UPSCMaterials https://telegram.me/FreeUPSCMaterials https://telegram.me/MaterialforExam

of goods. And because the system has more money, many will be willing to pay a higher price for the
same goods, leading to inflation. That is the theoretical argument.
 The point that is relevant is that more money in the banking system means higher bond prices. Why?
Because banks will have more money, they will buy more bonds.
 And that will push up bond prices. Since bonds carry an inverse price-yield relationship, higher bond
prices means lower yields.
 The problem is that lower yields will force the RBI to cut interest rates further.
 If the RBI does not want to cut rates, it will reduce (sterilise) liquidity that causes the yields to fall. It will
do so by selling the government bonds that it holds in its books. This means that sterilisation is possible
only to the extent that the RBI holds government bonds in its portfolio.
 This process of selling government bonds to reduce liquidity is part of its open market operations.

46. Ans: (d)


Explanation:
 Sunil Mehta committee on bad loans resolution has recommended a five-pronged strategy -- Project
'Sashakt' -- to deal with Non-performing Assets in the country's banking system. The five-pronged
strategy includes -- SME resolution approach, bank-led resolution approach, AMC/AIF led resolution
approach, NCLT/IBC approach, and asset-trading platform. The report recommends a five pronged
approach for loans from Rs 50 crore, Rs 50-500 crore and loans above Rs 500 crore.
 Reserve Bank of India (RBI) has issued a Prompt Corrective Action (PCA) framework to maintain sound
financial health of banks. It facilitates banks in breach of risk thresholds for identified areas of
monitoring, viz., capital, asset quality (which is tracked in terms of the net Non-Performing Assets ratio)
and profitability, to take corrective measures in a timely manner, in order to restore their financial
health. Thus, it is intended to encourage banks to eschew certain riskier activities, improve operational
efficiency and focus on conserving capital to strengthen them. The framework is not intended to
constrain the performance of normal operations of the banks for the general public. RBI has placed
eleven PSBs, viz., Dena Bank, Central Bank of India, Bank of Maharashtra, UCO Bank, IDBI Bank, Oriental
Bank of Commerce, Indian Overseas Bank, Corporation Bank, Bank of India, Allahabad Bank and United
Bank of India under the PCA framework.
 The Insolvency and Bankruptcy Code, 2016 (IBC) is the bankruptcy law of India which seeks to
consolidate the existing framework by creating a single law for insolvency and bankruptcy. The
bankruptcy code is a one stop solution for resolving insolvencies which at present is a long process and
does not offer an economically viable arrangement. A strong insolvency framework where the cost and
the time incurred is minimised in attaining liquidation has been long overdue in India. The code will be
able to protect the interests of small investors and make the process of doing business a less
cumbersome process.

47. Ans: (d)


Explanation:
 Deposit Insurance and Credit Guarantee Corporation (DICGC), a wholly owned subsidiary of Reserve
Bank of India was in operation since 1962 for this purpose.
 The Corporation insures all bank deposits, such as savings, fixed, current, and recurring.
 Deposit insurance extended by DICGC covers all commercial banks, including Local Area Banks (LABs)
and Regional Rural Banks (RRBs) in all the States and Union Territories (UTs). All Co-operative Banks
across the country except three UTs of Lakshadweep, Chandigarh, and Dadra and Nagar Haveli are also
covered by deposit insurance.
 There are some exceptions like deposits of foreign governments, deposits of Central/ State
Governments, deposits of State Land Development Banks with State co-operative banks, and inter-bank
deposits.
 The premium charged is on a flat rate basis which is 10 paise per Rs 100. The amount of coverage is
presently limited to Rs. 1 lakh.

48. Ans: (d)


Explanation:
https://telegram.me/UPSCMaterials https://telegram.me/FreeUPSCMaterials https://telegram.me/MaterialforExam

 Gini Coefficient is a popular statistical measure to gauge the rich-poor income or wealth divide. It
measures inequality of a distribution — be it of income or wealth — within nations or States. Its value
varies anywhere from zero to 1; zero indicating perfect equality and one indicating the perfect
inequality. Gini Coefficients can be used to compare income distribution of a country over time as well.
An increasing trend indicates that income inequality is rising independent of absolute incomes.
 A general rise in Gini Coefficient indicates that government polices are not inclusive and may be
benefiting the rich as much as (or even more than) the poor. For instance, a subsidy on passenger train
tickets may entail a big budget outlay and may be targeted at the poor. But its benefit could actually be
derived by the non-poor.
 Per capita income and Net Domestic product are the measures of national income, not the
development.
 The green gross domestic product (green GDP or GGDP) is an index of economic growth with the
environmental consequences of that growth factored into a country's conventional GDP. Green
GDPmonetizes the loss of biodiversity, and accounts for costs caused by climate change.

49. Ans: (a)


Explanation:
 The value of the currency notes and coins derived from the guarantee provided by the issuing authority of
these items.
 Every currency note bears on its face a promise from the Governor of RBI that if someone produces the
note to RBI, or any other commercial bank, RBI will be responsible for giving the person purchasing power
equal to the value printed on the note.
 These are called FIAT currency. Fiat money is a currency without intrinsic value that has been established
as money, often by government regulation. Fiat money does not have use value, and has value only
because a government maintains its value, or because parties engaging in exchange agree on its value.
 Commodity money is created from a good, often a precious metal such as gold or silver, which can be used
as a medium of exchange (such a good is called a commodity).

50. Ans: (a)


Explanation:
 Providing output based incentives to corporates will push them to produce more and exports more
which boosts the economy and yields revenue to the government. Currently our economy is driven by
domestic consumption.
 All other options like increasing the revenue expenditure, devaluing the currency and increasing the
salary of government employees will not lead to productive part as it simply incurs expenditure to
government without valuable production.

51. Ans: (a)


Explanation:
 Bank Rate refers to the official interest rate at which RBI will provide loans to the banking system.
 Bank rate and Repo Rate seem to be similar terms because in both of them RBI lends to the banks.
However, Repo Rate is a short-term measure and used for controlling the amount of money in the
market.
 Bank Rate is a long-term measure and is governed by the long-term monetary policies of the RBI.

52. Ans: (c)


Explanation:
 The banks may like to maximise their lending and keep their idle cash at a minimum so that profits are
higher. But if much of the funds are lent out and if there’s a sudden rush to withdraw, banks will struggle
to meet the repayments.
 It is to avoid this situation that the RBI specifies both a CRR and an SLR for banks.
 Only scheduled commercial banks and Cooperative banks needs to maintain CRR.
 When the SLR is reduced the money supply in the economy increases causing a rise in prices.
 SLR effects the money supply situation in economy which in turn affect the inflation of the economy.
https://telegram.me/UPSCMaterials https://telegram.me/FreeUPSCMaterials https://telegram.me/MaterialforExam

 Lower SLR, means bank can give more money as loan = lower interest rates = cheap loan = more people
take loan to start business or building house or buying car = boost in economy.
 Higher SLR, can curb inflation but may also lead to slowdown in economy

53. Ans: (d)


Explanation:
 Legal right to work: Unlike earlier employment guarantee schemes, the Act provides a legal right to
employment for adult members of rural households.
 At least one third beneficiaries have to be women.
 Wages must be paid according to the wages specified for agricultural labourers in the state under
the Minimum Wages Act, 1948, unless the central government notifies a wage rate (this should not be
less than Rs 60 per day).
 Time bound guarantee of work and unemployment allowance: Employment must be provided within 15
days of being demanded failing which an ‘unemployment allowance’ must be given.
 Decentralised planning: Gram sabhas must recommend the works that are to be undertaken and at
least 50% of the works must be executed by them.
 PRIs are primarily responsible for planning, implementation and monitoring of the works that are
undertaken.
 Work site facilities: All work sites should have facilities such as crèches, drinking water and first aid.
 Transparency and accountability: There are provisions for proactive disclosure through wall writings,
citizen information boards, Management Information Systems and social audits.
 Social audits are conducted by gram sabhas to enable the community to monitor the implementation of
the scheme.
 Funding: Funding is shared between the centre and the states. There are three major items of
expenditure – wages (for unskilled, semi-skilled and skilled labour), material and administrative costs.
 The central government bears 100% of the cost of unskilled labour, 75% of the cost of semi-skilled and
skilled labour, 75% of the cost of materials and 6% of the administrative costs.

54. Ans: (c)


Explanation:
 The Union Ministry of Petroleum & Natural Gas has announced new Open Acreage Licensing Policy
(OALP) for oil and gas exploration.
 The objective of OLAP is to increase India’s indigenous oil and gas production by maximising the
potential of already discovered hydrocarbon resources.
 Open Acreage Licensing Policy (OALP) gives an option to a company looking for exploring hydrocarbons
to select the exploration blocks on its own, without waiting for the formal bid round from the
Government. Under Open Acreage Licensing Policy (OALP), a bidder intending to explore hydrocarbons
like oil and gas, coal bed methane, gas hydrate etc., may apply to the Government seeking exploration of
any new block (not already covered by exploration).
 OALP was introduced vide a Cabinet decision of the Government dated 10.03.2016, as part of the new
fiscal regime in exploration sector called HELP or Hydrocarbon Exploration and Licensing Policy, so as to
enable a faster survey and coverage of the available geographical area which has potential for oil and gas
discovery.

55. Ans: (b)


Explanation:
 The Sunderban Reserve Forest, spread of 4,260 sq. km. with over 2,000 sq. km. of mangrove forests and
creeks, is likely to be declared a Ramsar Site soon. Earlier (April 2018), the West Bengal government gave
its approval to the State Forest Department to apply for recognition under the Ramsar Convention.
 Whereas currently East Kolkata wetlands in West Bengal is one of the wetlands of international
importance.
 The Indian Sunderbans, with 2,114 sq. km. of mangrove forests, comprise almost 43% of the mangrove
cover in the country according to a 2017 Forest Survey of India report. Other than the forests, home to
about 100 Royal Bengal tigers, the creeks and river systems of the Sunderbans are also part of the
https://telegram.me/UPSCMaterials https://telegram.me/FreeUPSCMaterials https://telegram.me/MaterialforExam

reserve forest and once conferred a Ramsar site status, it will be the largest protected wetland in the
country.

56. Ans: (c)


Explanation: Roles of Rbi as banker to government:
 Maintaining and operating of deposit accounts of the Central and State Governments.
 Receipts and collection of payments to the Central and State Governments.
 Making payments on behalf of the Central and State Governments.
 Transfer of funds and remittance facilities to the Central and State Governments.
 Managing the public debt and the issue of new loans and Treasury Bills of the Central Government.
 Providing ways and means advances to the Central and State Governments to bridge the interval
between expenditure and flow of receipts of revenue.
 Advising the Central and State Governments on financial matters, such as the quantum, timing and
terms of issue of new loans.
 The bank also tenders advice to the government on policies concerning banking and financial issues,
planning as well as resource mobilisation.
 The Reserve Bank represents the Government of India as member of the International Monetary Fund
and the World Bank.

57. Ans: (d)


Explanation:
 Treasury Bills are basically, instruments for short term (maturities less than one year) borrowing by the
Central Government.
 Treasury Bills or T-Bills are issued by the Central Government, Auctions of T-Bills are conducted by RBI.
 Individuals, Firms, Trusts, Institutions and banks can purchase T-Bills. The commercial and cooperative
banks use T-Bills for fulfilling their SLR requirements.
 T-Bills are issued on discount to face value, while the holder gets the face value on maturity. The return
on T-Bills is the difference between the issue price and face value. Hence the return depends upon the
auction value.

58. Ans: (b)


Explanation:
 Government securities are instruments issued by the government to borrow money from the market.
They are also known as gilts or gilt-edged securities.
Following features of government securities earned them the name of gilt-edged securities,
 They have zero income default
 There is high rate of return
 There is 100% liquidity
The other three option’s values depend upon their demand-supply, market sentiment, etc.Hence no assured
returns, they are subject to market risks.

59. Ans: (c)


Explanation:
 A trade deficit is an economic measure of international trade in which a country's imports exceeds its
exports. A trade deficit represents an outflow of domestic currency to foreign markets. It is also referred
to as a negative balance of trade (BOT).
 Trade Deficit = Total Value of Imports – Total Value of Exports.
 Since Rupee is being used for trading, the US dollar demand will come down and Rupee demand will
increase, since it is being used for more transactions. Therefore Rupee value will appreciate against US
dollar.
 Using our own currency improves the Balance of Payment status and therefore ability to pay for imports.
 For example, USA uses US dollars for trading, yet President DonaldTrump, is trying to narrow down trade
deficit with Europe, China, etc using import substitution and Tariff Barriers.
https://telegram.me/UPSCMaterials https://telegram.me/FreeUPSCMaterials https://telegram.me/MaterialforExam

60. Ans: (d)


Explanation: Direct Taxes in India:
 Income Tax
 Wealth Tax
 Gift Tax Act
 Personal Income Tax
 Capital Gains Tax
 Securities Transaction Tax
 Perquisite Tax
 Corporate Income Tax
 Tax on Agricultural Income

61. Ans: (b)


Explanation:
 All indirect taxes are regressive tax in nature, as it affect poor more than rich.
 Finance commission decides on the distribution of divisible taxes which includes both direct and indirect
taxes but not all of them.

62. ANS: (c)


Explanation: The most common cause of cost-push inflation starts with an increase in the cost of production
like increase in land rent, labour, raw materials cost and increase in taxes and transportation cost.

63. Ans: (a)


Explanation: Impact of increase in money supply
 Usually, an increase in the money supply will lead to a fall in interest rates. Lower interest rates will also
increase investment, economic activity.
 Capital formation increases as banks will have more money to lend.
 Socio-economic condition of the inhabitants increases. As people are left with more money (their
purchasing power increases).
 Inflation.

64. Ans: (b)


Explanation:
 National income (i.e, Net national product(NNP) at factor cost) is the monetary value of finished goods
and services produced by a country's citizens, overseas and domestically, in a given period.
 As in the first two statements no economic activity had happened, so it cannot be taken into account.

65. Ans: (d)


Explanation:
 The FAME India (Faster Adoption and Manufacture of Hybrid and Electric Vehicles) Scheme was
launched by the Ministry of Heavy Industries and Public Enterprises in 2015 to incentivize the production
and promotion of eco-friendly vehicles including electric vehicles and hybrid vehicles.
 FAME India is a part of the National Electric Mobility Mission Plan. Main thrust of FAME is to encourage
electric vehicles by providing subsidies. Vehicles in most segments – two wheelers, three wheelers,
electric and hybrid cars and electric buses obtained the subsidy benefit of the scheme.
 In early 2018, the Ministry of Power launched the new National Electric Mobility Programme with broad
objectives. Here, the focus is for establishing the electric charging infrastructure and a policy framework
to set realise more than 30% electric vehicles by 2030.

66. Ans: (A)


Explanation:
 For the conservation of Indus dolphins - one of the world’s rarest mammals - the Punjab government
along with WWF-India have conducted the first organised census on their population.
https://telegram.me/UPSCMaterials https://telegram.me/FreeUPSCMaterials https://telegram.me/MaterialforExam

 Found only in India and Pakistan, the Indus dolphins are confined to only a 185 km stretch between
Talwara and Harike Barrage in India’s Beas River in Punjab.

67. Ans: (A)


Explanation:
 Van Dhan Yojana, launched in Bijapur on April 14, involves setting up of skill upgradation and value
addition centres in tribal villages. These centres, also called Van Dhan Vikas Kendras, would first train
tribals in how to add value to minor forest produce they gather and then sell at throwaway prices. The
value addition would help in enhancing their income.
 It aims at mainstreaming tribal community by promoting primary level value addition to Non Timber
Minor Forest Produce (MFP) at grassroots level. It also aims to promote MFPs-centric livelihood
development of tribal gatherers and artisans. Through this initiative, the share of tribals in the value
chain of Non-Timber Forest Produce is expected to rise from present 20% to around 60%.

68. ANS: (D)


Explanation: Key features of the Payment Banks as per RBI Guidelines are:
i) Objectives:
The objectives of setting up of payments banks will be to further financial inclusion by providing (i) small
savings accounts and (ii) payments/remittance services to migrant labour workforce, low income
households, small businesses, other unorganised sector entities and other users.
ii) Eligible promoters:
a. Existing non-bank Pre-paid Payment Instrument (PPI) issuers; and other entities such as individuals /
professionals; Non-Banking Finance Companies (NBFCs), corporate Business Correspondents(BCs),
mobile telephone companies, super-market chains, companies, real sector cooperatives; that are
owned and controlled by residents; and public sector entities may apply to set up payments banks.
b. A promoter/promoter group can have a joint venture with an existing scheduled commercial bank to
set up a payments bank. However, scheduled commercial bank can take equity stake in a payments
bank to the extent permitted under Section 19 (2) of the Banking Regulation Act, 1949.
c. Promoter/promoter groups should be ‘fit and proper’ with a sound track record of professional
experience or running their businesses for at least a period of five years in order to be eligible to
promote payments banks.
iii) Scope of activities:
a. Acceptance of demand deposits. Payments bank will initially be restricted to holding a maximum
balance of Rs. 100,000 per individual customer.
b. Issuance of ATM/debit cards. Payments banks, however, cannot issue credit cards.
c. Payments and remittance services through various channels.
d. BC of another bank, subject to the Reserve Bank guidelines on BCs.
e. Distribution of non-risk sharing simple financial products like mutual fund units and insurance
products, etc.
iv) Deployment of funds :
a. The payments bank cannot undertake lending activities.
b. Apart from amounts maintained as Cash Reserve Ratio (CRR) with the Reserve Bank on its outside
demand and time liabilities, it will be required to invest minimum 75 per cent of its "demand deposit
balances" in Statutory Liquidity Ratio(SLR) eligible Government securities/treasury bills with maturity
up to one year and hold maximum 25 per cent in current and time/fixed deposits with other scheduled
commercial banks for operational purposes and liquidity management.
v) Capital requirement:
The minimum paid-up equity capital for payments banks shall be Rs. 100 crore.
a. The payments bank should have a leverage ratio of not less than 3 per cent, i.e., its outside liabilities
should not exceed 33.33 times its net worth (paid-up capital and reserves).
vi) Promoter's contribution: The promoter's minimum initial contribution to the paid-up equity capital of
such payments bank shall at least be 40 per cent for the first five years from the commencement of its
business.
vii) Foreign shareholding: The foreign shareholding in the payments bank would be as per the Foreign Direct
Investment (FDI) policy for private sector banks as amended from time to time.
https://telegram.me/UPSCMaterials https://telegram.me/FreeUPSCMaterials https://telegram.me/MaterialforExam

India post payment bank (IPPB) is the only Govt. owned payment bank. Other established payment banks in
India are Aditya Birla Idea, Airtel, Paytm, Fino and these are private owned.

69. Ans: (A)


Explanation:
SAMADHAN:
 Scheme of Asset Management and Debt Change Structure, or SAMADHAN, the bankers' consortium
shortlisted 11 power plants with an overall capacity of over 12 GW, which are either complete or nearing
completion.
 This scheme is an effort to avoid liquidation of these plants at throwaway considerations.
 State Bank of India and some banks have come together. They blew up a scheme called SAMADHAN.
Under this scheme, they will try to revive some assets to run.
PARIWARTAN:
The government plans to warehouse stressed power projects totalling 25,000 megawatts under an asset
management firm to protect the value of the assets and prevent their distress sale under the insolvency and
bankruptcy code till demand for power picks up.
 State-run Rural Electrification Corp. Ltd had identified projects with a total debt of around Rs 1.8 trillion
as part of the scheme, which is under government consideration and has been tentatively named Power
Asset Revival through Warehousing and Rehabilitation, or ‘Pariwartan’.
 The ‘Pariwartan’ scheme is inspired by the Troubled Asset Relief Programme, or TARP, which was
introduced in the US during the 2008 financial crisis.
 The proposed plan also aims to stem the rise in bad loans in the power sector.
 These stressed power projects will be housed under an asset management and Rehabilitation Company
that will be owned by financial institutions. The plan is being driven by concerns that stressed projects
have drawn bids for around Rs 1-2 crore per MW under the insolvency and bankruptcy code, a fraction
of the Rs 5 crore per MW needed to build them.
 While the promoter’s equity will be reduced to facilitate a transfer of management control to financial
institutions, the lenders will convert their debt into equity. The AMRC will manage the projects and may
ask utilities such as NTPC to operate and maintain them. The AMRC will charge a fee and help complete
projects that are stranded for lack of funds.

70. Ans: (c)


Explanation:
 What is GST?
 GST is one indirect tax for the whole nation, which will make India one unified common market.
 GST is a single tax on the supply of goods and services, right from the manufacturer to the
consumer.
 101st constitutional Amendment act
 Which taxes at the Centre and State level are being subsumed into GST?
At the Central level, the following taxes are being subsumed:
 Central Excise Duty,
 Additional Excise Duty,
 Service Tax,
 Additional Customs Duty commonly known as Countervailing Duty, and
 Special Additional Duty of Customs.
At the State level, the following taxes are being subsumed:
 Subsuming of State Value Added Tax/Sales Tax,
 Entertainment Tax (other than the tax levied by the local bodies), Central Sales Tax (levied by the
Centre and collected by the States),
 Octroi and Entry tax,
 Purchase Tax,
 Luxury tax, and
 Taxes on lottery, betting and gambling.
https://telegram.me/UPSCMaterials https://telegram.me/FreeUPSCMaterials https://telegram.me/MaterialforExam

 How will imports be taxed under GST?


 The Additional Duty of Excise or CVD and the Special Additional Duty or SAD presently being levied
on imports will be subsumed under GST. As per explanation to clause (1) of article 269A of the
Constitution, IGST will be levied on all imports into the territory of India. Unlike in the present
regime, the States where imported goods are consumed will now gain their share from this IGST paid
on imported goods.
 But Custom duty is exempted from the purview of GST.
 Article 279A empowers the GST Council to recommend to states and the union government “any special
rate or rates for a specified period, to raise additional resources during any natural calamity or disaster.”
A special provision in the Constitution—article 246 A-- also empowers every state legislature to make
laws with respect to goods and services tax.
 A proposal from a state to levy a special tax or a cess has to be discussed in the GST Council. A decision
will be guided by these constitutional provisions subject to the attorney general’s views on legality of a
cess.

71. Ans: (c)


Explanation:
 To measure inflation in an economy, usually, Wholesale Price Index (WPI) and Consumer Price Index (CPI)
are used. Wholesale Price Index helps in measuring the average change in prices received on bulk sale of
goods. On the other hand, Consumer Price Index is one that computes the changes in the general price
level of a class of consumer goods.
 CPI has a housing component which includes the change in real estate prices whereas WPI is only for
goods.
 Correlation of the price rise faced by common man was much better with CPI rather than WPI in the years
after 2011. The main reason for inflation experienced at the ground level was high food and fuel prices.
Food and fuel inflation have a higher weight in CPI when compared to WPI. Hence , the RBI shifted to CPI
for making all monetary policy decisions.
 Traditionally, the major reason for the CPI inflation being higher than WPI inflation has been that food
articles had a higher weight (48.3 per cent) in CPI than in WPI (24.3 per cent). This factor plays an
important role, whenever the primary trigger of inflation is food inflation. It is to be noted that food
inflation in the last few years is being spurred more by items such as vegetables, pulses and eggs, meat
and fish than cereals.

72. Ans: (b)


Explanation:
 The high-level committee headed by Sunil Mehta on restructuring stressed assets and creating more
value for public sector banks (PSBs) has suggested a transparent market-based solution with a focus on
asset turnaround to ensure job protection and creation.
 The committee has put forward PROJECT SASHAKT, a five pronged strategy to deal stressed assets.
 The resolution process suggested by the committee will also help bring in credible long-term external
capital to limit the burden on the domestic banking sector while ensuring robust governance and credit
architecture to prevent a similar build-up of non-performing loans in the future.
 The five-pronged resolution route — outlining an SME resolution approach, bank-led resolution
approach, AMC/AIF led resolution approach, NCLT/IBC approach, and asset-trading platform —
envisaged by the committee will be applicable to smaller assets with exposure up to >50 crore, mid-size
assets between >50 crore and >500 crore, and large assets with exposure of >500 crore and more which
have a potential for turnaround.

73. Ans: (A)


 The Government can sell its enterprises completely to the private sector or disinvest a part of its equity
capital held by it to the private sector companies or in the open market.
 Disinvestment involves the sale of Government’s share in the Public Sector Undertakings. In
disinvestment, the government sells only a part of the equity which is essentially less than 51% so that
ownership and management rights can be possessed by the Government itself.
https://telegram.me/UPSCMaterials https://telegram.me/FreeUPSCMaterials https://telegram.me/MaterialforExam

 The money obtained from such share selling through disinvestment will be kept with a special fund
called National Investment Fund (NIF). The NIF money will be utilized for specific purposes set by the
government. Working of the NIF since its inception indicate that in most occasions, the government has
utilized the disinvestment revenue in the NIF for capitalizing PSUs. But between the period of 2008 to
2013, bulk (around 75%) of the funds were used to finance social sector expenditure as the tax revenue
was inadequate to support these heads in the context of economic slowdown.
 From fiscal year 2013-14, the disinvestment proceeds will be credited to the existing NIF which is a
‘Public Account’ under the Government Accounts. The funds would remain there until
withdrawn/invested for the approved purposes.

74. Ans: (a)


Explanation:
 Intensive growth is when a firm grows by expanding its product line or its market reach. Thus, if a firm
introduces a new product, enters a new market, or further develops its own competency, than the firm
is undergoing intensive growth.
 Extensive growth, in economics, is growth in the quantity of output produced based on the expansion of
the quantity of inputs used. It contrasts with intensive growth, which arises from inputs being used more
productively.
 Inclusive growth is a concept that advances equitable opportunities for economic participants
during economic growth with benefits incurred by every section of society. This concept expands upon
traditional economic growth models to include focus on the equity of health, human capital,
environmental quality, social protection, and food security.
 Extensive growth, in economics, is growth in the quantity of output produced based on the expansion of
the quantity of inputs used. It contrasts with intensive growth, which arises from inputs being used more
productively. For example, GDP growth caused only by increases in population or territory would be
extensive growth. Thus, extensive growth is likely to be subject to diminishing returns. It is therefore
often viewed as having no effect on per-capita magnitudes in the long-run.

75. Ans: (c)


Explanation:
 The US has decided to review India’s eligibility to enjoy duty-free access for certain products in the
American market under a tax benefit scheme.
 As many as 3,500 Indian products from sectors such as chemicals and engineering get duty-free access to
the US market under the Generalized System of Preferences (GSP), introduced in 1976.
 Generalized System of Preferences (GSP) is a preferential tariff system extended by developed countries
(also known as preference giving countries or donor countries) to developing countries (also known as
preference receiving countries or beneficiary countries).
 It involves reduced MFN Tariffs or duty-free entry of eligible products exported by beneficiary countries
to the markets of donor countries.
Benefits of GSP:
 Indian exporters benefit indirectly - through the benefit that accrues to the importer by way of reduced
tariff or duty free entry of eligible Indian products
 Reduction or removal of import duty on an Indian product makes it more competitive to the importer -
other things (e.g. quality) being equal.
 This tariff preference helps new exporters to penetrate a market and established exporters to increase
their market share and to improve upon the profit margins, in the donor country.

76. Ans: (C)


Explanation:
 As of June 2018, OPEC has 15-member countries: six in the Middle East (Western Asia), seven in Africa,
and two in South America.
 The current OPEC members are: Algeria, Angola, Ecuador, Equatorial Guinea, Gabon, Iran, Iraq, Kuwait,
Libya, Nigeria, Qatar, the Republic of the Congo, Saudi Arabia (the de facto leader), United Arab
Emirates, and Venezuela. Indonesia is a former member.
https://telegram.me/UPSCMaterials https://telegram.me/FreeUPSCMaterials https://telegram.me/MaterialforExam

 By continent, two are South American, seven African, and six are Asian (Middle East). Two-thirds of
OPEC's oil production and reserves are in its six Middle-Eastern countries that surround the oil-rich
Persian Gulf.
 The 13 non-OPEC countries which are also oil producers, include: Russia, Azerbaijan, Bahrain, Brunei,
Equatorial Guinea, Kazakhstan, Malaysia, Mexico, Oman, Russia, Sudan, South Sudan, Brazil and Bolivia.

77. Ans: (A)


Explanation:
 The National Disaster Management Authority (NDMA) is conducting a basic training programme at the
Lok Nayak Jayaprakash Airport in Patna. The five-day training programme, aimed at enhancing the
preparedness of Airport Emergency Handlers (AEHs) to respond to CBRN emergencies at the airports.
 CBRN emergencies pertain to threats emanating due to Chemical, Biological, Radiological and Nuclear
material.
 NDMA involves with the following disasters
 Natural Disasters
 Earthquakes
 Floods
 Landslides
 Cyclones
 Tsunamis
 Urban Floods
 Heat Wave
 Man-Made Disasters
 Nuclear Disaster
 Chemical Disaster
 Biological Disaster

78. Ans: (b)


Explanation:
 Deflation is the general decline in prices for goods and services occurring when the inflation rate falls
below 0%. Deflation happens naturally when the money supply of an economy is fixed.
 Measures to control Deflation :
 Monetary measures :
 Lowering bank reserve limits
 Open market operations
 Lowering the target interest rates
 Quantitative easing
 Negative interest rates
 Fiscal Policy measures
 Increase government spending
 Cut tax rates

79. Ans: (b)


Explanation:
 Stagflation is a condition of slow economic growth and relatively high unemployment, or
economic stagnation, accompanied by rising prices, or inflation. It can also be defined as inflation and a
decline in gross domestic product (GDP). Stagflation is an economic problem defined in equal parts by its
rarity and by the lack of consensus among academics on how exactly it comes to pass.
 Usually, when unemployment is high, spending declines, as do the prices of goods. Stagflation occurs
when the prices of goods rise while unemployment increases and spending declines.
 Stagflation is also considered an unnatural phenomenon since inflation shouldn't happen when an
economy is weak. In most cases, weak or slower economic growth should prevent inflation from
happening.
https://telegram.me/UPSCMaterials https://telegram.me/FreeUPSCMaterials https://telegram.me/MaterialforExam

 Disinflation is a temporary slowing of the pace of price inflation. It is used to describe instances when
the inflation rate has reduced marginally over the short term. It should not be confused with deflation,
which can be harmful to the economy.
 Reflation is a fiscal or monetary policy, designed to expand a country's output and curb the effects
of deflation, which usually occurs after a period of economic uncertainty or a recession. As such, the
term "reflation" is also used to describe the first phase of economic recovery after a period of
contraction.

80. Ans: (a)


Explanation:
 Gross national product (GNP) is an estimate of total value of all the final products and services turned
out in a given period by the means of production owned by a country's residents. GNP is commonly
calculated by taking the sum of personal consumption expenditures, private domestic investment,
government expenditure, net exports and any income earned by residents from overseas investments,
minus income earned within the domestic economy by foreign residents. Net exports represent the
difference between what a country exports minus any imports of goods and services.
 Because it refers to the income available to residents as the result of their contribution to production, it
is appropriate for analyses related to sources and uses of income. For example, saving rates are normally
expressed as a percentage of income, and GNP is the more appropriate measure for this propose. In
addition, GNP is better than GDP for analyses that focus on the availability of resources, such as the
Nation’s ability to finance expenditures on education.
 Usually in developing countries, GDP is greater than GNP because those countries tend to get more
investment inflows rather than outflows, so the produce more inside the borders.

81. Ans: (c)


Explanation:
 Fiat money is currency that a government has declared to be legal tender, but it is not backed by a
physical commodity. The value of fiat money is derived from the relationship between supply and
demand rather than the value of the material from which the money is made.
 The fiat money will lose its value if people and institutions lose their confidence on a Government or Fiat
issuing authority in repaying the debts and managing the Financial health.
Consequences of Increase in Black money:
 Growth of economic dualism,
 Under-estimation of the true size of the economy,
 Tax evasion, thereby loss of revenue to Government,
 Undermining equity,
 Widening Gap between the Rich and Poor,
 Lavish Consumption Spending,
 Distortion of Production Pattern,
 Distribution of Scarce Resources,
 Deterioration of General Moral Standards of the Society,
 Effects on Production.

82. Ans: (d)


Explanation:
 Personal income refers to all income collectively received by all individuals or households in a country.
 Personal income includes compensation from a number of sources including salaries, wages and bonuses
received from employment or self-employment; dividends and distributions received from investments;
rental receipts from real estate investments and profit-sharing from businesses.
 Disposable income, also known as disposable personal income (DPI), is the amount of money that
households have available for spending and saving after income taxes have been accounted for.
 Disposable personal income = Personal income – Taxes.

83. Ans: (c)


https://telegram.me/UPSCMaterials https://telegram.me/FreeUPSCMaterials https://telegram.me/MaterialforExam

 Mutual Fund is trusts which pool the savings of large number of investors and then reinvests those funds
for earning profits and then distribute the dividend among the investors.
 The Mutual Funds usually invest their funds in equities, bonds, debentures, call money etc., depending
on the objectives and terms of scheme floated by MF. Nowadays there are MF which even invest in
gold or other asset classes.
 A mutual fund will provide diversification through the exposure to a multitude of stocks. The reason that
is recommended over owning a single stock is that owning an individual stock would carry more risk than
a mutual fund.
 Hedge funds are alternative investments using pooled funds that employ numerous different strategies
to earn active return, for their investors. Hedge funds may be aggressively managed or make use of
derivatives and leverage in both domestic and international markets with the goal of generating high. It
is important to note that hedge funds are generally only accessible to accredited investors as they
require less regulations than other funds. One aspect that has set the hedge fund industry apart is the
fact that hedge funds face less regulation than mutual funds and other investment vehicles. High value
return, High risks.

84 Ans: (c)
Explanation:
 Capital Receipts:
Government receipts which either (i) create liabilities (e.g. borrowing) or (ii) reduce assets (e.g.
disinvestment) are called capital receipts. Thus when govt. raises funds either by incurring a liability or
by disposing off its assets, it is called a capital receipt.
 Revenue Receipts:
 Non Tax revenue
 External grants-in-aid: Financial help from foreign governments and international organisations in
the form of grants, donations, gifts and contribution.

85. Ans: (a)


Explanation:
 Unnat Bharat Abhiyan is flagship programme of Ministry of Human Resource Development (HRD) that
aims to enrich Rural India. The second edition (Unnat Bharat Abhiyan 2.0) was launched in April, 2018.
Currently 748 institutions are participating under the scheme.
 The scheme aims to link higher education institutions with set of atleast five villages, so that they can
contribute to economic and social betterment of these village communities using their knowledge base.
Under this scheme, higher education institutions will participate in development activities, particularly in
rural areas.

86. Ans: (c)


Explanation:
Reporters Without Borders (RWB) is an international non-profit, non-governmental organization based in
Paris, France, that conducts political advocacy on issues relating to freedom of information and freedom of
the press. The Press Freedom Index is an annual ranking of countries compiled and published by Reporters
without Borders based upon the organisation's own assessment of the countries' press freedom records in
the previous year.

87. Ans: (c)


Explanation:
Earth Day is an annual event celebrated on April 22. Worldwide, various events are held to demonstrate
support for environmental protection. First celebrated in 1970, Earth Day events in more than 193 countries
are now coordinated globally by the Earth Day Network. Earth Day 2018 theme is 'End Plastic Pollution'.

88. Ans (c)


https://telegram.me/UPSCMaterials https://telegram.me/FreeUPSCMaterials https://telegram.me/MaterialforExam

 Commercial bank is a bank that is formed for the commercial purpose and hence its primary aim to earn
profit from the banking business. On the other hand, cooperative banks are owned and operated by the
members for a common purpose i.e. to provide financial service to agriculturists and small businessmen.
It relies on the on the principles of cooperation, such as open membership, democratic decision making,
mutual help.
 Commercial banks are under Banking Regulations Act 1949 while cooperative banks are under
Cooperative societies Act, 1965.
 Interest rates offered by cooperative banks are slightly higher than that offered by Commercial banks.
 Anyone can borrow in Commercial banks but only member shareholders can borrow in cooperatives.
 Recent guidelines from RBI stated that, Urban Cooperative Banks (UCB) should lend 40% of ANBC to
Priority sectors which is same as for commercial banks.

89. Ans: (b)


Explanation:
 Frictional Unemployment: The minimum amount of unemployment that prevails in an economy due to
workers quitting their previous jobs and are searching for the new jobs is called Frictional
Unemployment.
 Cyclical Unemployment: Cyclical unemployment is due to deficiency or fall in effective demand from
consumers which leads to fall in production and low demand for labour. Cyclical unemployment is a type
of unemployment which is related to the cyclical trends of booms and recessions called as the business
cycle. If an economy is doing good, cyclical unemployment will be at its lowest and will be the highest if
the economy faces recession.
 Voluntary Unemployment: Voluntary unemployment refers to a situation where workers are either not
seeking for work or are in transition from one job to another (quitting one job in search of another
better job).
 Structural unemployment: Structural unemployment refers to a situation which arises due to change in
the structure of the economy. Example: An economy transforms itself from a Labour intensive economy
to a Capital intensive economy. Structural unemployment usually occurs due to the mismatch of skills.
 Seasonal Unemployment: Seasonal unemployment occurs during certain seasons of the year. In some
industries and occupations like agriculture, holiday resorts etc., production activities take place only in
some seasons.
 Disguised Unemployment: Disguised unemployment is when too many people are employed than what
is required to produce efficiently. This kind of employment is not at all productive. It is not productive in
a sense that production does not suffer even if some of the employed people are withdrawn. The key
point to remember is that the marginal productivity of labourers under disguised unemployment is zero.
The labourers are employed physically, but not economically.

90. Ans: (b)


Explanation:
 Revenue deficit is the gap between the consumption expenditure (revenue expenditure) of the
Government (Union or the State Governments) and its current revenues (revenue receipts)
 The gross fiscal deficit (GFD) is the excess of total expenditure including loans net of recovery over
revenue receipts (including external grants) and non-debt capital receipts. The net fiscal deficit is the
gross fiscal deficit less net lending of the Central government.
 So if the ratio increases then it means that the government spends the borrowed amount for non-
productive revenue purposes, like paying employees’ salary, giving subsidy under PDS, etc.

91. Ans: (a)


Explanation: Self explanatory

92. Ans: (c)


 Monetary policy implies those measures designed to ensure an efficient operation of the economic
system or set of specific objectives through its influence on the supply, cost and availability of money.
 Objectives of Monetary Policy :
https://telegram.me/UPSCMaterials https://telegram.me/FreeUPSCMaterials https://telegram.me/MaterialforExam

 Price stability or control of inflation


 Exchange rate stability
 Economic growth
 Other options are the objectives of fiscal policy by the Government.

93. Ans: (a)


 The money multiplier is the amount of money that banks generate with each dollar of reserves.
Reserves is the amount of deposits that the Federal Reserve requires banks to hold and not lend.
Banking reserves is the ratio of reserves to the total amount of deposits.
 The multiplier effect is basically the amount of cash that banks generate with each dollar of reserves, or
the ratio of deposits compared to reserves that are circulating in the banking system. That means
increasing the amount of money in the money supply by taking in deposits, keeping some in reserves (an
amount prescribed by the central bank) and lending out the rest.

94. Ans: (d)


Explanation: Self explanatory

95. Ans: (a)


Explanation:
 Project Dhoop urges schools to shift their morning assembly to noon time, mainly between 11 am and 1
pm to ensure maximum absorption of Vitamin D in students through natural sunlight.
 The Food Safety and Standards Authority of India (FSSAI) launched a nationwide campaign today (April 9)
at National Bal Bhawan in Delhi to spread awareness about availing Vitamin D through natural sunlight
and consuming fortified food among school-going children.
 Food Safety and Standards Authority of India (FSSAI) is an autonomous body established under the
Ministry of Health & Family Welfare, Government of India. The FSSAI has been established under the
Food Safety and Standards Act, 2006 which is a consolidating statute related to food safety and
regulation in India. FSSAI is responsible for protecting and promoting public health through the
regulation and supervision of food safety.

96. Ans: (b)


Explanation:
 The World Economic Outlook (WEO) is a survey conducted and published by the International Monetary
Fund. It is published biannually and partly updated two times a year. It portrays the world economy in
the near and medium context, with projections for up to four years into the future.
 The Global Financial Stability Report (GFSR) is a semi annual report by the International Monetary Fund
(IMF) that assesses the stability of global financial markets and emerging-market financing. It is released
twice per year, in April and October. The GFSR focuses on current conditions, especially financial and
structural imbalances, that could risk an upset in global financial stability and access to financing by
emerging-market countries.
 Global Economic Prospects is a World Bank Group Flagship Report that examines global economic
developments and prospects, with a special focus on emerging market and developing countries, on a
semi annual basis (in January and June). The January edition includes in-depth analyses of topical policy
challenges faced by these economies, while the June edition contains shorter analytical pieces.

97. Ans: (b)


Explanation:
 The overall goal of the GIAHS Programme is to identify and safeguard Globally Important Agricultural
Heritage Systems and their associated landscapes, agricultural biodiversity and knowledge systems
through catalyzing and establishing a long-term programme to support such systems and enhance
global, national and local benefits derived through their dynamic conservation, sustainable management
and enhanced viability.
 To achieve this goal, the main objectives are:
https://telegram.me/UPSCMaterials https://telegram.me/FreeUPSCMaterials https://telegram.me/MaterialforExam

1. To leverage global and national recognition of the importance of agricultural heritage systems and
institutional support for their safeguard;
 Global recognition is obtained through the creation of the Agricultural Heritage Systems
categories supported by governments, FAO governing bodies, UNESCO, World Heritage Centre
and other partners:
 National recognition and awareness is raised by improving understanding of the threats that
such agricultural systems face, of their global importance and of the benefits that they provide
at all levels.
2. Capacity building of local farming communities and local and national institutions to conserve and
manage GIAHS, generate income and add economic value to goods and services of such systems in a
sustainable fashion;
 Identify ways to mitigate risks of erosion of biodiversity and traditional knowledge, land
degradation and threats posed by globalization processes, and skewed policies and incentives;
 Strengthen conservation and sustainable use of biodiversity and natural resources, reducing
vulnerability to climate change, enhancing sustainable agriculture and rural development and as
a result contributing to food security and poverty alleviation;
 Enhancing the benefits derived by local populations from conservation and sustainable use of
their resources and their ingenious systems and rewarding them through the payment for
environmental services, eco-labelling, eco-tourism and other incentive mechanisms and market
opportunities.
3. To promote enabling regulatory policies and incentive environments to support the conservation,
evolutionary adaptation and viability of GIAHS;
 Assessment of existing policies and incentive mechanisms, and identification of modalities to
provide support for sustainable agricultural practices;
 Promotion of national and international processes leading to improved policies and incentive
mechanisms.

98. Ans: (c)


Explanation:
 As part of Shanghai Cooperation Organization (SCO) initiatives, SCO Peace Mission Exercise is conducted
biennially for SCO member states.
 The joint exercise for the year 2018 was conducted by Central Military Commission of Russia from 22
August to 29 August 2018 at Chebarkul, Chelyabinsk, Russia.
 This will be a historic occasion due to the maiden participation of India post becoming a full member of
the SCO in June 2017.
 For the first time, the militaries of India and Pakistan are taking part in this mega anti-terror drill of the
Shanghai Cooperation Organisation (SCO) in Russia aimed at expanding cooperation among the member
countries to deal with the growing menace of terrorism and extremism.

99. Ans (d)


Explanation:
 The Union Ministry of Culture for the first time had organised Madhavpur Mela in Madhavpur Ghed in
Porbandar District of Gujarat under the banner of Ek Bharat Shrestha Bharat. Its purpose is to bring
various parts of country especially the North-East, close to each other.
 The Nabakalebara is an ancient ritual associated with most of the Jagannath Temples when the Idols of
Lord Jagannath, Balabhadra, Subhadra and Sudarshan are replaced by a new set of Idols. This festival is
celebrated at the Jagannath Temple in Puri, Orissa.
 Manipur's much-awaited five-day Shirui Lily State Festival, in the Ukhrul district headquarters. The
festival, organized by the State Tourism Department, aims to spread awareness and conserve the
endangered state flower ‘Shirui Lily’, while also promote the hill district as a potential tourist
destination.
 The Shirui Hills is known for a unique species of land-lily, the Shirui Lily which grows at a height of about
8,500 ft above sea level. It was declared as the State Flower of Manipur in 1989. The flower is today
https://telegram.me/UPSCMaterials https://telegram.me/FreeUPSCMaterials https://telegram.me/MaterialforExam

considered an endangered species. The Shirui Hills is considered one of the most interesting treks in
Manipur.

100. Ans: (d)


Explanation:
 Market Stabilization scheme (MSS) is a monetary policy intervention by the RBI to withdraw excess
liquidity (or money supply) by selling government securities in the economy. The MSS was introduced in
April 2004. Main thing about MSS is that it is used to withdraw excess liquidity or money from the
system by selling government bonds.
 The issued securities are government bonds and they are called as Market Stabilisation Bonds (MSBs).
Thus, the bonds issued under MSS are called MSBs. These securities are owned by the government
though they are issued by the RBI. It is to be remembered that government is the owner of the
securities.
 The money obtained under MSS should be kept with the RBI. It should not be transferred to the
government. This is because, if it is transferred, government will spend the money in the economy
thereby adding to liquidity.
 As per the latest policy, the government has increased the amount of MSBs to be issued to Rs 6 lakh
crores from just 0.3 lakh crores in the context of demonetisation.

Anda mungkin juga menyukai